next up previous
Next: Solution to Problem 2. Up: Solutions to Final Exam Previous: Solutions to Final Exam


Solution to Exercise 1.

(1) [20 points] Suppose $X$ and $Y$ are independent random variables with the distributions as given below. For each case, determine the distribution of $Z$ $=$ $XY$ either by giving the cdf, pdf, or pmf. (a) $X$ is uniform(0,1) and $Y$ is exponential ($\mu$).

Solution: Since the joint distribution of $(X,Y)$ is continuous, we work with the pdf

\begin{displaymath}
f_{XY} (x,y) \; = \; I_{(0,1)} (x) \mu^{-1} \exp[-y/\mu] I_{(0,\infty)} (y) ,
\end{displaymath}

and apply the ``Jacobian'' theory. The transformation we will use is

\begin{displaymath}
(W,Z) \; = \; (Y,XY) \quad \Longrightarrow \quad (X,Y) \; = \; (Z/W,W) .
\end{displaymath}

The Jacobian is

\begin{displaymath}
\left\vert \frac{\partial (x,y)}{\partial (w,z)} \right\ver...
... (-z/w^2) \cdot 0 - (1/w) \cdot 1 \right\vert
\; = \; 1/w .
\end{displaymath}

Substituting in the joint pdf, we have

\begin{eqnarray*}
f_{WZ} (w,z) & \; = \; & f_{XY} (z/w,w) \cdot (1/w) \\
& \;...
...{(0,1)} (z/w) \mu^{-1} w^{-1} \exp[-w/\mu] I_{(0,\infty)} (w) .
\end{eqnarray*}

Note that $I_{(0,1)} (z/w) I_{(0,\infty)} (w)$ is $1$ if and only if $0 < z/w < 1$ $\Longleftrightarrow$ $0 < z < w < \infty$. For our purposes, we note that

\begin{displaymath}
I_{(0,1)} (z/w) I_{(0,\infty)} (w) \; = \; I_{(z,\infty)} (w) I_{(0,\infty)} (z) .
\end{displaymath}

Now we want the marginal pdf $f_Z (z)$, which is
$\displaystyle f_Z (z)$ $\textstyle \; = \;$ $\displaystyle \int \, f_{WZ} (w,z) \, dw$  
  $\textstyle \; = \;$ $\displaystyle \int_z^{\infty} \, \mu^{-1} w^{-1} \exp[-w/\mu] \, dw I_{(0,\infty)} (z)$ (1)
    $\displaystyle \mbox{ change variables } v = w/z$  
  $\textstyle \; = \;$ $\displaystyle \mu^{-1} \int_{1}^{\infty} \, v^{-1} \exp[-vz/\mu] \, dw I_{(0,\infty)} (z)$  
  $\textstyle \; = \;$ $\displaystyle \mu^{-1} \mbox{Ei}(1,z/\mu) I_{(0,\infty)} (z) ,$  

where $\mbox{Ei}(n,y)$ $=$ $\int_1^{\infty} x^{-n} \exp[-yx] dx$ is a well known special function called the Exponential Integral. (What? You don't know this special function? No problem. I didn't count off as long as you got a correct integral expression for $f_Z (z)$ as in (1) or (2) below.) An alternative expression (if you chose the transformation $(W,Z) \; = \; (X,XY)$ is
\begin{displaymath}
f_Z (z) \; = \; \int_0^1 \, (\mu w)^{-1} \exp \left[ -z / (w \mu) \right] \, dw
I_{(0,\infty)} (z) .
\end{displaymath} (2)




(b) $X$ is uniform(0,1) and $Y$ is Bernoulli(1/2).

Solution: Here, $X$ has a continuous distribution but $Y$ is discrete. We can't use a Jacobian argument here. Also, we expect that the distribution of $Z = XY$ might be a mixture of discrete and continuous, so we won't have a pdf or a pmf. It will be necessary to give the distribution using the cdf. Clearly, $Y=0$, which occurs with probability $1/2$, means $Z = 0$. If $Y = 1$, then $Z = X$. Thus, $P[ Z = 0] = 1/2$, and if $0 < z < 1$, then
$\displaystyle F_Z (z)$ $\textstyle \; = \;$ $\displaystyle P[ Z \le z ]$  
  $\textstyle \; = \;$ $\displaystyle P[ Z \le z \vert Y = 0 ] P[ Y = 0 ] \, + \,
P[ Z \le z \vert Y = 1 ] P[ Y = 1 ]$  
  $\textstyle \; = \;$ $\displaystyle 1 \cdot (1/2) \, + \, P[ X \le z ] \cdot (1/2)$  
  $\textstyle \; = \;$ $\displaystyle (1/2)(1+z) .$ (3)

Putting it together, we have
\begin{displaymath}
F_z (z) \; = \; \left\{ \begin{array}{ccl}
0 & & \mbox{i...
... \le z \le 1\\
1 & & \mbox{if } z > 1.
\end{array} \right.
\end{displaymath} (4)




(c) $X$ is Poisson(1) and $Y$ is Bernoulli(1/2).

Solution: Both $X$ and $Y$ are discrete, so we don't use any Jacobians. The distribution of $Z$ will be discrete with possible values $\{ 0, 1 , \ldots \}$, the nonnegative integers. The pmf is

\begin{eqnarray*}
f_Z (z) & \; = \; & P[ Z = z ] \\
& \; = \; &
P[Z=z\vert Y...
...& & \mbox{if } z \in \{ 1 , 2 , \ldots \}.
\end{array} \right.
\end{eqnarray*}


next up previous
Next: Solution to Problem 2. Up: Solutions to Final Exam Previous: Solutions to Final Exam
Dennis Cox 2003-01-18